You are on page 1of 11

2 RESISTIVE CIRCUITS

2.1 Ohm’s Law


2.2 Nodes, Branches, and Loops
2.3 Kirchhoff’s Laws
2.4 Series Resistors and Voltage Division
2.5 Parallel Resistors and Current Division
2.6 Wye-Delta Transformations

2.1 OHM’S LAW

• Ohm’s law states that the voltage across a resistor is directly proportional
to the current I flowing through the resistor.

• Mathematical expression for Ohm’s Law is as follows:

v=iR

• Two extreme possible values of R: 0 (zero) and  (infinite) are related


with two basic circuit concepts: short circuit and open circuit.

• Conductance is the ability of an element to conduct electric current; it is


the reciprocal of resistance R and is measured in mhos or siemens.
1 i
G 
R v

• The power dissipated by a resistor:


v2
p  vi  i 2 R 
R

2-1
2.2 NODES, BRANCHES AND LOOPS

• A branch represents a single element such as a voltage source or a resistor.


• A node is the point of connection between two or more branches.
• A loop is any closed path in a circuit.
• A loop is said to be independent if it contains a branch which is not in any
other loop. Independent loops or paths result in independent sets of
equations.

Example 2.1

How many branches, nodes and loops are there in the circuit of Figure 2.1 (a)?

Figure 2.1 (a) Original circuit

Figure 2.1 (b) Equivalent circuit

Solution:

Since there are five elements in the circuit, the circuit has five branches: 10 V,
5 Ω, 2 Ω, 3 Ω, and 2A.

The circuit has three nodes as identified : node a, node b, and node c.
There are six loops in the circuit:
Loop a-b-c-a : Independent loop (Include branches of 10 V, 5 Ω, 2 Ω)
Loop b-c-b : Independent loop (Include branches of 2 Ω, 3 Ω)
Loop b-c-b : Independent loop (Include branches of 3 Ω, 2 A)
Loop a-b-c-a : Dependent loop (Include branches of 10 V, 5 Ω, 3 Ω)
Loop b-c-b : Dependent loop (Include branches of 2 Ω, 2 A)
Loop a-b-c-a : Dependent loop (Include branches of 10 V, 5 Ω, 2 A)

2-2
Exercise 2.1

How many branches, nodes and loops are there in the circuit of Figure 2.2?

Should we consider it as one branch or two


branches?

Figure 2.2

2.3 KIRCHHOFF’S LAWS

 Kirchhoff’s current law (KCL)


 States that the algebraic sum of currents entering a node (or a
closed boundary) is zero.
Mathematically, KCL implies that,
N

i
n 1
n 0

 To use KCL, an algebraic sign corresponding to a reference


direction must be assigned to every current at the node.
 The sum of the currents entering a node is equal to the sum of the
currents leaving the node.

 Kirchhoff’s voltage law (KVL)


 States that the algebraic sum of all voltages around a closed path
(or loop) is zero.
N

v
n 1
n 0

where v n = the nth voltage, N = number or voltages in the loop


 To use KVL, assign an algebraic sign (reference direction) to each
voltage in the loop.
 The sum of voltage drops is equal to the sum of voltage rises.

2-3
Example 2.2

Write down the KCL equation for circuit in Figure 2.3.

Solution:

Applying KCL gives


i1  (i2 )  i3  i4  (i5 )  0

By rearranging the terms, we get


i1  i3  i4  i2  i5

Figure 2.3

Example 2.3

Write down the KVL equation for circuit in Figure 2.4.

Solution:

Applying KVL gives


 v1  v2  v3  v4  v5  0

By rearranging the terms, we get


v2  v3  v5  v1  v4

Figure 2.4

Exercise 2.2

Write down the KVL equation for circuit in Figure 2.5(a). Calculate i, υ1 and υ2
for Figure 2.5(b).

Figure 2.5

2-4
2.4 SERIES RESISTORS AND VOLTAGE DIVISION

• Series: Two or more elements are in series if they are cascaded or


connected sequentially and consequently carry the same current.

• The equivalent resistance of any number of resistors connected in a series


is the sum of the individual resistances.
N
Req  R1  R2      RN   Rn
n 1

• Consider the single loop circuit of Figure 2.6, the two resistors are in
series, since the same current i flows in both of them.
• Applying Ohm’s law to each of the resistors, we obtain
v1  iR1 and v2  iR2 (i)
• If we apply KVL to the loop (in clockwise direction), we have
 v  v1  v2  0 (ii)
• Combining equation (i) and (ii) , v  i( R1  R2 ) (iii)
• To determine voltage across each resistor in Figure 2.6, substitute equation
(iii) into equation (i) and obtain
R1 R2
v1  v , v2  v (iv)
R1  R2 R1  R2
• From equation (iv) it is shown that, the source voltage υ is divided among
the resistor in direct proportion to their resistance. This is called the
principle of voltage division.
• The larger the resistance, the larger the voltage drop.
• In general, if the voltage divider has n resistors, the voltage drop of nth
resistor can be expressed as

Rn
vn  v
R1  R2      RN

Figure 2.6 Voltage divider circuit

2-5
2.5 PARALLEL RESISTORS AND CURRENT DIVISION

• Parallel: Two or more elements are in parallel if they are connected to the
same two nodes and consequently have the same voltage across them.

• The equivalent resistance of a circuit with N resistors in parallel is:


1 1 1 1
    
Req R1 R2 RN
• For parallel resistance, the total current i is shared by the resistors in
inverse proportion to their resistances. This is known as the principle of
current division. Notice that the larger current flows through the smaller
resistance.

Figure 2.7 Current divider circuit

 Refer to circuit in Figure 2.7, based on KCL


i  i1  i2 ……..(i)
V V
Based on Ohm’s law, i1  1 & i2  2 …….(ii)
R1 R2
By substituting (ii) into (i)
V V
i 1  2 where V  V1  V2
R1 R2
1 1 
i V  
 R1 R2 
 R  R2 
i V 1  ……(iii)
 R1 R2 
From (iii)
i ( R1 R2 )
V ……(iv)
( R1  R2 )

Substituting (iv) into (ii)


R2 R1
i1  i and i2  i
R1  R2  R1  R2 

2-6
2.6 WYE-DELTA TRANSFORMATIONS

Delta -> Star Star -> Delta

R1 R2  R2 R3  R3 R1
R1 
Rb Rc Ra 
( Ra  Rb  Rc ) R1

Rc Ra R1 R2  R2 R3  R3 R1
R2  Rb 
( Ra  Rb  Rc ) R2

Ra Rb R1 R2  R2 R3  R3 R1
R3  Rc 
( Ra  Rb  Rc ) R3

2-7
Tutorial 2 : Resistive Circuits

Q1. The voltage across a 5-kΩ resistor is 16V. Find the current through the
resistor.

Q2. Find the hot resistance of a light bulb rated 60W, 120V.

Q3. When the voltage across a resistor is 120V, the current through it is
2.5mA. Calculate its conductance.

Q4. (a) Calculate current i in Figure Q4 when the switch in position 1.


(b) Find the current when the switch is in position 2.

1 2

i
100 3V 150

Figure Q4

Q5. Determine v1 through v4 in the circuit in Figure Q5.

 
v1  v2
12 V
 

 8V   6V 


 
10V
v4 v3
  

Figure Q5

2-8
Q6. For the circuit in Figure Q6, use KCL to find the branch currents I1 to I4.

2A

I2 7A I4

I1 3A I3 4A

Figure Q6

Q7. Find v1 and v2 in the circuit in Figure Q7.

 v1 

v1
6V 
12V 10V

 v2 

Figure Q7

Q8. Calculate the power dissipated in the 5-Ω resistor in the circuit of Figure
Q8.

1
v 
o

45V
 3v o

5

Figure Q8

2-9
Q9. In the circuit in Figure Q9, find v, i, and the power absorbed by the 4-Ω
resistor.

5 4
i

20V v 10  6

Figure Q9

Q10. Calculate the equivalent resistance Rab at terminals a-b in Figure Q10.

10 
a

80 
60  20  30 
b

Figure Q10

Q11. Find Req at terminals a-b in Figure Q11.

70 
a
30 
40 
60 
b
20 

Figure Q11

Q12. Obtain the equivalent resistance at the terminals a-b for the circuit in
Figure Q12.

a
10  20 
30 
10 
10  20 
b

Figure Q12

2-10
Answers :

Q1. i = 3.2mA Q7. v1 = 14V, v2 = 22V


Q2. R = 240 Ω Q8. P = 1125W
Q3. G = 20.833 μS Q9. v = 10V, i = 1A, P = 4W
Q4. (a) i = 30 mA (b) i = 20 mA Q10. Rab= 16 
Q5. v1 = -4V, v2 = -6V, v3 = 4V, Q11. Req= 76 
v4 = -2V
Q6. I1 = 12A, I2 = -10A, I3 = 5A, Q12. Rab= 9.23 
I4 = -2A

2-11

You might also like